Cyclically 130

Algebra Level 5

Consider all quadruples of real numbers ( x 1 , x 2 , x 3 , x 4 ) , (x_1,x_2,x_3,x_4), satisfying x 1 x 2 x 3 x 4 x_1\leq x_2 \leq x_3 \leq x_4 , such that

{ x 1 + x 2 x 3 x 4 = 130 x 2 + x 3 x 4 x 1 = 130 x 3 + x 4 x 1 x 2 = 130 x 4 + x 1 x 2 x 3 = 130 \begin{cases} x_1 + x_2x_3x_4 & = 130\\ x_2 + x_3x_4x_1 & = 130\\ x_3 + x_4x_1x_2 & = 130\\ x_4 + x_1x_2x_3 & = 130\\ \end{cases}

If S S is the sum of all possible values of x 4 , x_4, what is S , S, rounded to the nearest integer ?

Details and assumptions

Each possible distinct value of x 4 x_4 should be counted only once, even if it appears in several different quadruples.


The answer is 395.

This section requires Javascript.
You are seeing this because something didn't load right. We suggest you, (a) try refreshing the page, (b) enabling javascript if it is disabled on your browser and, finally, (c) loading the non-javascript version of this page . We're sorry about the hassle.

9 solutions

Mark Hennings
Oct 22, 2013

The equations tell us that ( x 1 x 2 ) ( 1 x 3 x 4 ) = ( x 3 x 4 ) ( 1 x 1 x 2 ) = 0 (x_1-x_2)(1-x_3x_4) \; = \; (x_3-x_4)(1-x_1x_2) \; = \; 0 There are four cases to consider:

  1. If x 1 = x 2 = a x_1=x_2 = a and x 3 = x 4 = b x_3 = x_4 = b then a + a b 2 = b + a 2 b = 130 a + ab^2 = b + a^2b = 130 , and hence ( a b ) ( 1 a b ) = 0 (a-b)(1-ab) = 0 . If a = b a=b then x 1 = x 2 = x 3 = x 4 = 5 x_1=x_2=x_3=x_4=5 . If a b = 1 ab=1 then a + b = 130 a+b=130 and hence x 1 = x 2 = 65 8 66 x_1=x_2 = 65-8\sqrt{66} , x 3 = x 4 = 65 + 8 66 x_3=x_4=65+8\sqrt{66} .

  2. If x 1 = x 2 x_1 = x_2 and x 1 x 2 = 1 x_1x_2=1 then either x 1 = x 2 = 1 x_1=x_2=1 , in which case x 3 + x 4 = 130 x_3+x_4=130 and x 3 x 4 = 129 x_3x_4 = 129 , so that x 3 = 1 x_3=1 and x 4 = 129 x_4=129 , or else x 1 = x 2 = 1 x_1=x_2=-1 , in which case x 3 + x 4 = 130 x_3+x_4 = 130 and x 3 x 4 = 131 x_3x_4 = -131 , so that x 3 = 1 x_3=-1 and x 4 = 131 x_4=131 .

  3. If x 3 x 4 = 1 x_3x_4=1 and x 3 = x 4 x_3=x_4 then either x 3 = x 4 = 1 x_3=x_4=1 , in which case x 1 + x 2 = 130 x_1+x_2=130 and x 1 x 2 = 129 x_1x_2 = 129 , so that x 1 , x 2 = 1 , 129 x_1,x_2 = 1,129 , or else x 3 = x 4 = 1 x_3=x_4 = -1 , in which case x 1 + x 2 = 130 x_1+x_2=130 and x 1 x 2 = 131 x_1x_2=-131 , so that x 1 , x 2 = 1 , 131 x_1,x_2=-1,131 . Neither of these cases is possible.

  4. If x 3 x 4 = x 1 x 2 = 1 x_3x_4 = x_1x_2 = 1 then x 1 + x 2 = x 3 + x 4 = 130 x_1+x_2 = x_3+x_4= 130 , and hence x 1 = x 3 = 65 8 66 x_1=x_3 = 65 - 8\sqrt{66} , x 2 = x 4 = 65 + 8 66 x_2=x_4 = 65 + 8\sqrt{66} , which is not possible.

Thus the sum of the possible values of x 4 x_4 is 5 + 65 + 8 66 + 129 + 131 = 394.992307 5 + 65 + 8\sqrt{66} + 129 + 131 \; = \; 394.992307 which is 395 395 to the nearest integer.

Nice solution! Did anyone notice how close this is to an integer? This is really not a coincidence.

Alexander Borisov - 7 years, 7 months ago

Nice solution! I have another way to arrive at the cases: when we multiply each equation by their "alone" variable, we get: x 1 2 130 x 1 = x 2 2 130 x 2 = x 3 2 130 x 3 = x 4 2 130 x 4 = x 1 x 2 x 3 x 4 x_1^2-130x_1=x_2^2-130x_2=x_3^2-130x_3=x_4^2-130x_4=-x_1x_2x_3x_4 . If the set of x 1 , x 2 , x 3 , x 4 x_1,x_2,x_3,x_4 form a three or four element set(three or four distinct numbers), then this would contradict the fact the a quadratic equation can have at most two distinct solutions.

Xuming Liang - 7 years, 3 months ago

Nice solution. I have one mathematical problem related to the differential equation. Can you help for that? my gmail address is jinjaladinesh@gmail.com.

Dinesh Jinjala - 3 years, 5 months ago

If x i = 0 x_i = 0 , then we must have x j + 0 = 130 x_j+0 = 130 for j i j \neq i . But then, we must also have 13 0 3 = x i + j i x j = 130 130^3 = x_i+ \prod_{j \neq i} x_j = 130 which is impossible. Hence x i 0 x_i \neq 0 for all i i . Let R = x 1 x 2 x 3 x 4 R = x_1x_2x_3x_4 , the given condition in the problem means x i + R x i = 130 x_i + \frac{R}{x_i} = 130 x i 2 130 x i + R = 0 \Rightarrow x_i^2 - 130x_i +R = 0 since x i 0 x_i \neq 0 . Now for i < j i < j , we have x i 2 130 x i + R = x j 2 130 x j + R x_i^2 -130x_i + R = x_j^2-130x_j +R ( x i x j ) ( x i + x j 130 ) = 0 (x_i-x_j)(x_i+x_j-130) = 0 Thus, if we set i = 1 i = 1 , we get to the conclusion that either x j = x 1 x_j = x_1 or x j = 130 x 1 x_j = 130-x_1 for j > 1 j > 1 . Divide into cases :

i) x 1 = x 2 = x 3 = x 4 = a x_1 = x_2 = x_3 = x_4 = a . Then we have the equation a + a 3 = 130 a+a^3 = 130 ( a 5 ) ( a 2 + 5 a + 26 ) = 0 (a-5)(a^2+5a+26)=0 The quadratic equation doesn't have real roots since the discriminant is negative. x 4 = 5 x_4 = 5 is the only solution for this case.

For the next cases x 4 = 130 x 1 > x 1 x_4 = 130-x_1 > x_1 , and so x 1 < 65 x_1 < 65

ii) x 1 < x 2 = x 3 = x 4 = 130 a x_1 < x_2 = x_3 = x_4 = 130-a , then we have the equation a + ( 130 a ) 3 = 130 a + a ( 130 a ) 2 = 130 a+(130-a)^3 = 130-a+a(130-a)^2 = 130 From the first equality, we have ( a ( 130 a ) ) + ( 130 a ) 2 ( ( 130 a ) a ) = 0 (a-(130-a))+(130-a)^2((130-a)-a) = 0 ( 2 a 130 ) ( 1 ( 130 a ) 2 ) = 0 (2a-130)(1-(130-a)^2) = 0 2 ( a 65 ) ( 1 + ( 130 a ) ) ( 1 ( 130 a ) ) = 0 2(a-65)(1+(130-a))(1-(130-a)) = 0 2 ( a 65 ) ( a 131 ) ( a 129 ) = 0 2(a-65)(a-131)(a-129) = 0 We know a 65 a \neq 65 , so we either have a = 131 130 a = 1 < a a = 131 \Rightarrow 130-a = -1 < a or a = 129 130 a = 1 < a a = 129 \Rightarrow 130-a = 1 < a . No solutions for this case

iii) a = x 1 = x 2 = x 3 < x 4 = 130 a a = x_1 = x_2 = x_3 < x_4 = 130a . By letting b = 130 a b = 130-a , we arrive to the same set of equations as case ii), and we get b = 129 a = 1 b = 129 \Rightarrow a = 1 which gives us ( x 1 , x 2 , x 3 , x 4 ) = ( 1 , 1 , 1 , 129 ) (x_1,x_2,x_3,x_4) = (1,1,1,129) and b = 131 a = 1 b = 131 \Rightarrow a = -1 which gives us ( x 1 , x 2 , x 3 , x 4 ) = ( 1 , 1 , 1 , 131 ) (x_1,x_2,x_3,x_4) = (-1,-1,-1,131) . We can easily check these two solutions are valid and the sum of all possible x 4 x_4 in this case 129 + 131 = 260 129+131 = 260

iv) a = x 1 = x 2 < x 3 = x 4 = 130 a a = x_1 = x_2 < x_3 = x_4 = 130-a , then we have the equation a + a ( 130 a ) 2 = 130 a + a 2 ( 130 a ) = 130 a+a(130-a)^2 = 130-a+a^2(130-a) = 130 From the first equality, we have ( a ( 130 a ) ) + a ( 130 a ) ( ( 130 a ) a ) = 0 (a-(130-a))+a(130-a)((130-a)-a) = 0 ( 2 a 130 ) ( 1 + a ( 130 a ) ) = 0 (2a-130)(1+a(130-a)) = 0 2 ( a 65 ) ( a 2 130 a + 1 ) = 0 2(a-65)(a^2-130a+1) = 0 Again we know that a 65 a \neq 65 , for the quadratic equation, let the roots be a 1 < a 2 a_1 < a_2 . By Vieta's theorem, a 1 + a 2 = 130 a_1 + a_2 = 130 and a 1 a 2 = 1 a_1a_2 =1 so both are them are positive (easy to check they are real numbers). Also we have a i ( 130 a i ) = 1 a_i(130-a_i) = 1 Using the above's equality implies a i + a i ( 130 a i ) 2 = a i + ( 130 a i ) = 130 a_i+a_i(130-a_i)^2 = a_i + (130-a_i) = 130 and also ( 130 a i ) + a i 2 ( 130 a i ) = ( 130 a i ) + a i = 130 (130-a_i)+a_i^2(130-a_i) = (130-a_i) + a_i = 130 and so a 1 , a 2 a_1,a_2 satisfies the equation. But only a 1 < 65 a_1 < 65 since their sum is 130 130 . By quadratic roots formula, a 1 = 65 6 5 2 1 = 1 65 + 6 5 2 1 < 0.5 a_1 = 65-\sqrt{65^2-1} = \frac{1}{65+\sqrt{65^2-1}} < 0.5 and so x 4 = 130 a i ( 129.5 , 130 ) x_4 = 130-a_i \in (129.5,130) . Since this is the last case and there is no other non-integer solutions in other cases, the sum for this case is 130 130

Hence, the grand total is 5 + 260 + 130 = 395 5+260+130 = 395 .

This is not the most efficient solution, but it is very nicely written.

Common mistakes included being somewhat ambiguous about using the symmetry of the equations, disregarding the condition that the variables x i are increasing. While it is possible to do so, one has to word this carefully. One way is to say something like this: "We will find all real solutions of the system, regardless of the order of the numbers, and will then pick x 4 to be the largest of the four numbers").

Calvin Lin Staff - 7 years ago
Tushar Gautam
May 20, 2014

Let x1=a : x2=b : x3=c : x4=d

a+bcd=130 ......(1)

b+cda=130 ......(2)

c+adb=130 ......(3)

d+abc=130 ......(4)

from 1 & 2 : a+bcd = b+cda i.e. (a-b)(1-cd) = 0

Case 1: a=b from 3 & 4 : (c-d)(1-ab)=0 i.e. (c-d)(1-a^2)=0

Case 1.1 : (1-a^2)=0 i.e. a= 1, -1

Case 1.1.1 : a=b=1 ; cd=129, c+d=130 ; c+129/c=130 ; c^2-130c+129=0 ; c=1,129 ; a=b=c=1, d=129 ;

Case 1.1.2 : a=b=-1 ; cd=-131, c+d=130 ; c-131/c=130 ; c^2-130c-131=0 ; c=-1, 131 ; a=b=c=-1, d=131 ;

Case 1.2 : c=d ;                  a(1+c^2)=130 ;                  c(1+a^2)=130 ;                  (a-c) - ac(a-c) = 0 ;                  (a-c)(1-ac) = 0 ;

Case 1.2.1 : a=c means a=b=c=d=5

Case 1.2.2 : ac=1 ; also, a(1+c^2)=130 ; so, c^2-130c+1=0 ; i.e. c=d=65+8 sqrt(66) , a=b=65-8 sqrt(66) ;

Case 2: cd=1 ; (3) c-(4) d ; (c-d)(c+d-130)=0

Case 2.1: c=d reduces to case 1.1

Case 2.2: c+d=130 ; ie c+1/c=130 ; ie c^2-130c+1=0 ; so, c=65-8 sqrt(66), d=65+8 sqrt(66) ; a+b=130 ; 65+8 sqrt(66) + ab (65-8 sqrt(66) ) = 130 ; ab=1 ; So, this reduces to Case 1.2.2 ;

So, in total, we have 4 solutions: a=b=c=-1, d=131 a=b=c=1, d=129 a=b=c=d=5 a=b=65-8 sqrt(66), c=d=65+8 sqrt(66)

So, S=131+129+5+65+8*sqrt(66) =394.9923 =395 (round off)

Solution 3 on QUora http://www.quora.com/Mathematics/Consider-all-quadruples-of-real-numbers-x1-x2-x3-x4-satisfying-x1≤x2≤x3≤x4-such-that-for-each-of-the-four-numbers-adding-it-to-the-product-of-the-other-three-numbers-equals-130-If-S-is-the-sum-of-all-possible-values-of-x4-what-is-S

Calvin Lin Staff - 7 years ago
Luuk Weyers
Oct 21, 2013

If x 4 = 0 x_4=0 , then x 1 = x 2 = x 3 = 130 x_1=x_2=x_3=130 . But 0 + 130 × 130 × 130 130 0+130 \times 130 \times 130 \neq 130 . So none of the x i x_i can be 0 0 .

Now we can safely multiply each equation with its 'lone' x x to get: ( 1 ) x 1 2 + x 2 x 3 x 4 = 130 x 1 (1) \, x_1^2+x_2x_3x_4=130x_1 ( 2 ) x 2 2 + x 1 x 3 x 4 = 130 x 2 (2) \, x_2^2+x_1x_3x_4=130x_2 ( 3 ) x 3 2 + x 1 x 2 x 4 = 130 x 3 (3) \, x_3^2+x_1x_2x_4=130x_3 ( 4 ) x 4 2 + x 1 x 2 x 3 = 130 x 4 (4) \, x_4^2+x_1x_2x_3=130x_4

( 4 ) ( 1 ) , ( 4 ) ( 2 ) , (4)-(1), (4)-(2), and ( 4 ) ( 3 ) (4)-(3) give us: x 1 = x 4 x 1 = 130 x 4 x_1=x_4 \vee x_1=130-x_4 x 2 = x 4 x 2 = 130 x 4 x_2=x_4 \vee x_2=130-x_4 x 3 = x 4 x 3 = 130 x 4 x_3=x_4 \vee x_3=130-x_4

There are now three different cases: All x i = x x_i=x , three of the x i = x x_i=x and the fourth equals 130 x 130-x , and two of the x i = x x_i=x and the other two equal 130 x 130-x .

The first case, x 1 = x 2 = x 3 = x 4 = x x_1=x_2=x_3=x_4=x , gives us: x + x 3 130 = 0 x+x^3-130=0 ( x 5 ) ( x 2 + 5 x + 26 ) = 0 (x-5)(x^2+5x+26)=0 x = 5 x = 5 + ι 2 x = 5 ι 2 x=5 \vee x= -\frac{5+\iota}{2} \vee x=-\frac{5-\iota}{2} x = 5 x=5 gives us x 1 = x 2 = x 3 = x 4 = 5. x_1=x_2=x_3=x_4=5.

The second case, x 1 = x 2 = x 3 = 130 x 4 = x x_1=x_2=x_3=130-x_4=x , gives us: 130 x + x 3 130 = 0 130-x+x^3-130=0 x 3 = x x^3=x x = 1 x = 0 x = 1 x=-1 \vee x=0 \vee x=1 x = 1 x=-1 gives us x 1 = x 2 = x 3 = 1 x_1=x_2=x_3=-1 and x 4 = 131 x_4=131 .

x = 1 x=1 gives us x 1 = x 2 = x 3 = 1 x_1=x_2=x_3=1 and x 4 = 129 x_4=129 .

The third case, 130 x 1 = 130 x 2 = x 3 = x 4 = x 130-x_1=130-x_2=x_3=x_4=x , gives us: 130 x + ( 130 x ) x 2 130 = 0 130-x+(130-x)x^2-130=0 x 3 130 x 2 + x = 0 x^3-130x^2+x=0 x = 0 x 2 130 x 2 + x = 0 x=0 \vee x^2-130x^2+x=0 x = 0 x = 65 6 5 2 1 x = 65 + 6 5 2 1 x=0 \vee x=65-\sqrt{65^2-1} \vee x=65+\sqrt{65^2-1} If x 3 = x 4 = 65 6 5 2 1 x_3=x_4=65-\sqrt{65^2-1} , x 1 = x 2 = 65 + 6 5 2 1 > x 4 x_1=x_2=65+\sqrt{65^2-1} > x_4 , so x 4 = 65 + 6 5 2 1 x_4=65+\sqrt{65^2-1} is the only viable solution in this case.

We have x 4 = 5 , 129 , 131 , x_4=5, 129, 131, or 65 + 6 5 2 1 65+\sqrt{65^2-1} .

5 + 129 + 131 + 65 + 6 5 2 1 394.99 5+129+131+65+\sqrt{65^2-1} \approx 394.99 . So the answer to the question is 395 \fbox{395} .

Nice solution!

Alexander Borisov - 7 years, 7 months ago
Nandita .
May 20, 2014

assume S = sum of all distinct values x4 can take

For ease of typing let me refer to x1, x2,x3,x4as a,b,c,and d.

Now it is given that:

  1. a+bcd = 130

  2. b+acd = 130

  3. c+abd = 130

  4. d+abc = 130

Multiplying 1 by a and 2 by b and subtracting leads to

a^2-b^2 = 130(a-b)

Therefore either a+b=130 or a=b

Taking other combination of equations (other than 1 and 2) and performing similar operations would lead to a symmetric result of

b+c=130 or b=c

c+d=130 or c=d

a+d=130 or a=d

Using a=b, (Using a=b also covers using b=c, c=d etc. by symmetry)

The equations reduce to a+acd=130, c+a^2d=130 and d+a^2c=130

The last two equations again lead to c=d or a^2=1

c=d => a+ac^2=130 and c+ca^2 = 130 which gives either a=c (giving a=b=c=d ) or ac=1 which along with the now simplified a+c=130 gives

a=b=129.992307, c=d=0.0076927 or a=b=0.0076927 and c=d=129.992307

Solution set = (129.992,129.992,0.00769,0.00769)

d being the highest value

a^2=1 => c+d=130 and (cd=129 from a=1 or cd= -131 from a= -1 )

leading to the solution sets (1,1,1,129) and (-1,-1,-1,131)

Not using a=b, we have a+b=130, b+c=130, c+d=130 and a+d=130 which gives a=b=c=d.

Finally the case a=b=c=d reduces the equations to a+a^3=130

The only real root of which is a=5.

Therefore S= 129.992307 +129 +131 + 5 = 394.9923

Solution 2 on Quora (http://www.quora.com/Mathematics/Consider-all-quadruples-of-real-numbers-x1-x2-x3-x4-satisfying-x1≤x2≤x3≤x4-such-that-for-each-of-the-four-numbers-adding-it-to-the-product-of-the-other-three-numbers-equals-130-If-S-is-the-sum-of-all-possible-values-of-x4-what-is-S)

"Not using a=b, we have a+b=130, b+c=130, c+d=130 and a+d=130 which gives a=b=c=d." Not true as stated: we may have a=c, b=d, and a+b=130. However there are more equations there than just those listed. However, overall, the solution is correct, just not perfectly written. Would be +2X if not for the cheating.

Calvin Lin Staff - 7 years ago
Tejas Kasetty
May 20, 2014

a+bcd=130 ......(1) b+cda=130 ......(2) c+adb=130 ......(3) d+abc=130 ......(4)

from 1 & 2 : a+bcd = b+cda i.e. (a-b)(1-cd) = 0

Case 1: a=b from 3 & 4 : (c-d)(1-ab)=0 i.e. (c-d)(1-a^2)=0

Case 1.1 : (1-a^2)=0 i.e. a= 1, -1

Case 1.1.1 : a=b=1 cd=129, c+d=130 c+129/c=130 c^2-130c+129=0 c=1,129 a=b=c=1, d=129

Case 1.1.2 : a=b=-1 cd=-131, c+d=130 c-131/c=130 c^2-130c-131=0 c=-1, 131 a=b=c=-1, d=131

Case 1.2 : c=d a(1+c^2)=130 c(1+a^2)=130 (a-c) - ac(a-c) = 0 (a-c)(1-ac) = 0

Case 1.2.1 : a=c means a=b=c=d=5

Case 1.2.2 : ac=1 also, a(1+c^2)=130 so, c^2-130c+1=0 i.e. c=d=65+8 sqrt(66) , a=b=65-8 sqrt(66)

Case 2: cd=1 (3) c-(4) d (c-d)(c+d-130)=0

Case 2.1: c=d reduces to case 1.1

Case 2.2: c+d=130 ie c+1/c=130 ie c^2-130c+1=0 so, c=65-8 sqrt(66), d=65+8 sqrt(66) a+b=130 65+8 sqrt(66) + ab (65-8 sqrt(66) ) = 130 ab=1 So, this reduces to Case 1.2.2

So, in total, we have 4 solutions: a=b=c=-1, d=131 a=b=c=1, d=129 a=b=c=d=5 a=b=65-8 sqrt(66), c=d=65+8 sqrt(66)

So, S=131+129+5+65+8*sqrt(66) =394.9923

Solution 3 on Quora. http://www.quora.com/Mathematics/Consider-all-quadruples-of-real-numbers-x1-x2-x3-x4-satisfying-x1≤x2≤x3≤x4-such-that-for-each-of-the-four-numbers-adding-it-to-the-product-of-the-other-three-numbers-equals-130-If-S-is-the-sum-of-all-possible-values-of-x4-what-is-S

Calvin Lin Staff - 7 years ago
Patrick Narciso
May 20, 2014

Let's consider the products

Multiplying the first equation by and the second equation by , and comparing, you get

Quite clearly, either , or , for any two indices.

Next, we show that you can't have three of the terms being different from each other, i.e. we can't have and at the same time. This is because if , then , and implies that . Clearly, we must have

This reduces the problem to 4 cases:

  1. This is a straightforward cubic equation, and we get

  2. 2.

This gives two simultaneous equations:

But don't forget that we also require , and that is the biggest of the lot. Substituting this requirement, you'll find that y = 1 and , or y = -1, .

    3.

Again, substituting in our requirement, we get .

  1. You'll find that this is identical to case 2, except that there won't be any solution with being the largest.

So the final answer that you need is

Solution 1 on Quora: http://www.quora.com/Mathematics/Consider-all-quadruples-of-real-numbers-x1-x2-x3-x4-satisfying-x1≤x2≤x3≤x4-such-that-for-each-of-the-four-numbers-adding-it-to-the-product-of-the-other-three-numbers-equals-130-If-S-is-the-sum-of-all-possible-values-of-x4-what-is-S

Calvin Lin Staff - 7 years ago
Shubham Raj
May 20, 2014

I am using a,b,c and d for the notations, we need to find sum of all possible values of d. we have a+ bcd=130 ...(1) b + acd=130...(2) c +abd =130...(3) d+ abc =130...(4)

Subtracting (2) from (1), a-b + cd(b-a)=0 (b-a)(cd-1) =0

Similarly, taking difference of any 2 equations from (1),(2), (3) and (4) We will get the following eqns :

(b-a)(cd-1)=0 (c-b)(ad-1)=0 (d-c)(ab-1)=0 (c-a)(bd-1)=0 (d-b)(ac-1)=0 (d-a)(bc-1)=0

One obvious solution is then a=b=c=d. (Case 1) Another will be a=b=c (case 2) Another will be a=b and c=d. (case 3)

Using these 3 cases and the first 4 eqns, possible values of d are 129,131,5 and 130(approx)

This is just a sketch of a solution, many details are missing.

Calvin Lin Staff - 7 years ago
Stella Marie
May 20, 2014

It is given that: 1. a+bcd = 130 2. b+acd = 130 3. c+abd = 130 and 4. d+abc = 130

Multiplying 1 by a and 2 by b and subtracting leads to a^2-b^2 = 130(a-b)

Therefore either a+b=130 or a=b

Taking other combination of equations (other than 1 and 2) and performing similar operations would lead to a symmetric result of

b+c=130 or b=c c+d=130 or c=d a+d=130 or a=d

Using a=b, (Using a=b also covers using b=c, c=d etc. by symmetry)

The equations reduce to a+acd=130, c+a^2d=130 and d+a^2c=130

The last two equations again lead to c=d or a^2=1

c=d => a+ac^2=130 and c+ca^2 = 130 which gives either a=c (giving a=b=c=d ) or ac=1 which along with the now simplified a+c=130 gives

a=b=129.992307, c=d=0.0076927 or a=b=0.0076927 and c=d=129.992307

Solution set = (129.992,129.992,0.00769,0.00769)

d being the highest value

a^2=1 => c+d=130 and (cd=129 from a=1 or cd= -131 from a= -1 )

leading to the solution sets (1,1,1,129) and (-1,-1,-1,131)

Not using a=b, we have a+b=130, b+c=130, c+d=130 and a+d=130 which gives a=b=c=d.

Finally the case a=b=c=d reduces the equations to a+a^3=130

The only real root of which is a=5.

Therefore S= 129.992307 +129 +131 + 5 = 394.9923

Solution 2 on Quora

Calvin Lin Staff - 7 years ago

0 pending reports

×

Problem Loading...

Note Loading...

Set Loading...